Đến nội dung

Hình ảnh

Topic tổng hợp một số bất đẳng thức trong kì thi MO các nước

* * * * * 16 Bình chọn

  • Please log in to reply
Chủ đề này có 501 trả lời

#281
Nguyenhuyen_AG

Nguyenhuyen_AG

    Trung úy

  • Thành viên nổi bật 2016
  • 945 Bài viết

  Bài 72 (IMO 2005): Cho các số thực dương $a,b,c$ thỏa mãn $abc\geq 1$. CMR:

 

      $\frac{a^5-a^2}{a^5+b^2+c^2}+\frac{b^5-b^2}{b^5+c^2+a^2}+\frac{c^5-c^2}{c^5+a^2+b^2}\geq 0$

 

Lời giải đặc biệt của kỳ thi.

 

Ta có

\[\frac{a^5-a^2}{a^5+b^2+c^2}-\frac{a^5-a^2}{a^3(a^2+b^2+c^2)}=\frac{(a-1)^2(a^2+a+1)(b^2+c^2)}{a(a^5+b^2+c^2)(a^2+b^2+c^2)} \ge 0.\]

Suy ra

\[\sum \frac{a^5-a^2}{a^5+b^2+c^2} \ge \sum\frac{a^5-a^2}{a^3(a^2+b^2+c^2)} = \sum \frac{a^2-\frac{1}{a}}{a^2+b^2+c^2} \ge \sum \frac{\sum a^2-\sum bc}{a^2+b^2+c^2} \ge 0.\]
---------------

@ducvipdh12: chào mừng sự trở lại của anh Huyện,lâu lắm mới thấy anh onl diễn đàn :))


Bài viết đã được chỉnh sửa nội dung bởi ducvipdh12: 16-06-2015 - 10:19

Nguyen Van Huyen
Ho Chi Minh City University Of Transport

#282
Namthemaster1234

Namthemaster1234

    Thiếu úy

  • Thành viên
  • 550 Bài viết

Bài 108: (APMO 2004):

 

Cho a,b,c là 3 số thực không âm. Chứng minh: $(a^2+2)(b^2+2)(c^2+2)\geqslant 9(ab+ac+bc)$

$\prod (a^2+2)=a^2b^2c^2+4(\sum a^2)+2(\sum a^2b^2)+8=(a^2b^2c^2+1+1)+\sum 2(a^2b^2+1)+4(\sum a^2)\geqslant \frac{9abc}{a+b+c}+(a^2+b^2+c^2)+7(ab+ac+bc)\geqslant 2(ab+ac+bc)+7(ab+ac+bc)$ (Theo Schur )


Đừng lo lắng về khó khăn của bạn trong toán học, tôi đảm bảo với bạn rằng những khó khăn toán học của tôi còn gấp bội.
(Albert Einstein)

Visit my facebook: https://www.facebook.com/cao.simon.56

:icon6: :icon6: :icon6: :icon6: :icon6:


#283
Hoang Long Le

Hoang Long Le

    Thượng sĩ

  • Thành viên
  • 265 Bài viết

$\prod (a^2+2)=a^2b^2c^2+4(\sum a^2)+2(\sum a^2b^2)+8=(a^2b^2c^2+1+1)+\sum 2(a^2b^2+1)+4(\sum a^2)\geqslant \frac{9abc}{a+b+c}+(a^2+b^2+c^2)+7(ab+ac+bc)\geqslant 2(ab+ac+bc)+7(ab+ac+bc)$ (Theo Schur )

 

Bài 108: (APMO 2004):

 

Cho a,b,c là 3 số thực không âm. Chứng minh: $(a^2+2)(b^2+2)(c^2+2)\geqslant 9(ab+ac+bc)$

 Ta chứng minh bất đẳng thức mạnh hơn đó là $(a^2+2)(b^2+2)(c^2+2)\geqslant 3(a+b+c)^2$

 Áo dụng C-S ta có : $3(a+b+c)^2\leq 3(a^2+2)\left [ 1+\frac{(b+c)^2}{2} \right ]$

 Ta cần chứng minh $2(b^2+2)(c^2+2)\geq 3[2+(b+c)^2]\Leftrightarrow 2(bc-1)^2+(b-c)^2\geq 0$ luôn đúng

 


IM LẶNG

#284
Hoang Nhat Tuan

Hoang Nhat Tuan

    Hỏa Long

  • Thành viên
  • 974 Bài viết

Bài 108: (APMO 2004):

 

Cho a,b,c là 3 số thực không âm. Chứng minh: $(a^2+2)(b^2+2)(c^2+2)\geqslant 9(ab+ac+bc)$

Câu này có một cách sử dụng Dirichlet

Ta có: $9\sum ab\leq 3(\sum a)^2$

Cần chứng minh: $\prod (a^2+2)\geq 3(\sum a)^2$

Theo Dirichlet giả sử $(b^2-1)(c^2-1)\geq 0$

Ta có: $(b^2+2)(c^2+2)=3(b^2+c^2+1)+(b^2-1)(c^2-1)\geq 3(b^2+c^2+1)$

$3(a^2+1+1)(1+b^2+c^2)\geq 3(a+b+c)^2$

=> ĐPCM :D


Ngài có thể trói cơ thể tôi, buộc tay tôi, điều khiển hành động của tôi: ngài mạnh nhất, và xã hội cho ngài thêm quyền lực; nhưng với ý chí của tôi, thưa ngài, ngài không thể làm gì được.

#285
Nguyenhuyen_AG

Nguyenhuyen_AG

    Trung úy

  • Thành viên nổi bật 2016
  • 945 Bài viết

Bài 40:
Cho $x,y,z$ là các số thực dương thỏa mãn $x+y+z=xyz$.CMR

$(x-1)(y-1)(z-1)\leq 6\sqrt{3}-10$


Ta chỉ cần xét trường hợp $x-1 > 0,\, y-1 > 0,\, z-1>0.$ Đặt $a = x-1 > 0,\, b = y-1 > 0,\, c = z-1>0,$ khi đó $x=a+1,\,y=b+1,\,z=c+1,$ giả thiết của bài toán trở thành
\[abc+ab+bc+ca=2.\]
Đặt $t=\sqrt[3]{abc},$ áp dụng bất đẳng thức AM-GM ta có
\[2 = abc+ab+bc+ca = t^3+ab+bc+ca \geqslant t^3 + 3\sqrt[3]{{ab}\cdot{bc}\cdot{ca}}=t^3+3t^2.\]
Do $t > 0,$ nên từ bất phương trình $2 \geqslant t^3+3t^2$ ta được $t \leqslant \sqrt{3}-1,$ suy ra $t^3 \leqslant \left(\sqrt{3}-1\right)^3.$ Suy ra tiếp
\[(x-1)(y-1)(z-1) = t^3 \leqslant \left(\sqrt{3}-1\right)^3 = 6\sqrt{3}-10.\]
Bài toán được chứng minh.

Bài viết đã được chỉnh sửa nội dung bởi Nguyenhuyen_AG: 17-06-2015 - 02:17

Nguyen Van Huyen
Ho Chi Minh City University Of Transport

#286
Nguyenhuyen_AG

Nguyenhuyen_AG

    Trung úy

  • Thành viên nổi bật 2016
  • 945 Bài viết

Bài 55:(Ailen 2009)

Cho $a,b,c$ là các số thực thỏa $\left\{\begin{matrix} a+b+c=0\\a^2+b^2+c^2=1 \end{matrix}\right.$

Chứng minh rằng

$a^2b^2c^2\leq \frac{1}{54}$

 

Bài này hồi đấy anh chế như vầy. :D

 

Xét bất đẳng thức hiển nhiên sau

\[(a-b)^2(b-c)^2(c-a)^2 \geqslant 0. \quad (1)\]

Đặt $p=a+b+c,\,q=ab+bc+ca,\,r=abc.$ Khi đó khai triển $(1)$ ta được

\[p^2q^2-4q^3+2p(9q-2p^2)r-27r^2 \geqslant 0.\]

Cho $p = a+b+c=0,$ thì

\[-4q^3-27r^2 \geqslant 0,\]

hay

\[4(ab+bc+ca)^3+27a^2b^2c^2 \leqslant 0. \quad (2)\]

Do $a+b+c=0$ nên $a^2+b^2+c^2=(a+b+c)^2-2(ab+bc+ca)=-2(ab+bc+ca),$ do đó bất đẳng thức $(2)$ trở thành

\[(a^2+b^2+c^2)^3 \geqslant 54a^2b^2c^2.\]

Cho $a^2+b^2+c^2=1,$ ta được

\[a^2b^2c^2 \leqslant \frac{1}{54}.\]

Từ đó có bài toán trên. :D


Nguyen Van Huyen
Ho Chi Minh City University Of Transport

#287
Nguyenhuyen_AG

Nguyenhuyen_AG

    Trung úy

  • Thành viên nổi bật 2016
  • 945 Bài viết

Mình xin nhận xét một vài điều như sau :

III. Bài 75 : Cách giải của nhungviekimcuong rất hay :) . Cách mình cũng không khác cách của bạn ấy nhưng mình xin phân tích bài giải của mình . 

Thực sự , khi gặp các dạng bài như dạng bài 75 , mình cố gằng tìm dấu $=$ nhưng lại không thấy do đó minh chợt nghĩ tới ý tưởng của phương pháp SS với các bđt lệch tâm bằng cách chứng minh một bđt mạnh hơn mà giá trị tại dấu $=$ là tối thiểu . Sau khi thử nhiều số thì nhận thấy với bộ cặp số $(-3,1,1)$ làm cho giá trị $VT-VP$ có vẻ như là nhỏ nhất và đẹp nhất :P . Thế nên không do dự , dựa vào kết quả thừa  mình cộng thêm vế phải một đại lượng $\frac{4}{7}(x+y+z)^{4}$ để bđt xảy ra khi (-3,1,1) và các hoán vị . Nên ta cần chứng minh : $(a+b)^{4}+(b+c)^{4}+(c+a)^{4}\geq \frac{4}{7}(a^{4}+b^{4}+c^{4}+(a+b+c)^{4})$

Tiếp nối ý tưởng của  nhungvienkimcuong  , đặt $\left\{\begin{matrix} a+b=x & & \\ b+c=y & & \\ c+a=z & & \end{matrix}\right.$

Khai triển và rút gọn (Trâu bò đấy :P ) ta được : $24(\sum x^{4})\geq 24\sum (xy)^{2}$ (luôn đúng) (Có thể phân tích sai :) )

 

Bất đẳng thức

\[(a+b)^{4}+(b+c)^{4}+(c+a)^{4}\geq \frac{4}{7}(a^{4}+b^{4}+c^{4}+(a+b+c)^{4}), \quad (2.9.1)\]

được anh Cẩn đề xuất rất lâu trên Mathlinks và được nhắc đến trong quyển Cauchy-Schwarz anh Cẩn. Ta có một chứng minh đơn giản sau đây.

 

Nếu $a+b+c=0,$ thay $c=-a-b$ vào (2.9.1) và thu gọn ta được

\[a^4+2a^3b+3a^2b^2+2ab^3+b^4\ge 0,\]

ta có

\[a^4+2a^3b+3a^2b^2+2ab^3+b^4 = (a^2+ab+b^2)^2 \ge 0.\]

Nếu $a+b+c\ne 0$, thay $(a,\, b,\, c)$ bởi $(-a,\, -b,\, -c)$ thì bất đẳng thức vẫn không thay đổi nên ta có thể giả sử $a+b+c>0,$ và chuẩn hóa cho $a+b+c=3.$

Đặt $a=x+1,\,b=y+1,\,c=z+1,$ thì $x+y+z=0$ và $x^3+y^3+z^3=3xyz,$ ta có

$$(a+b)^4+(b+c)^4+(c+a)^4 = x^4+y^4+z^4+24(x^2+y^2+z^2)-24xyz+48,$$

tương tự thì

$$a^4+b^4+c^4+(a+b+c)^4 = x^4+y^4+z^4+6(x^2+y^2+z^2)+12xyz+84.$$

Như vậy bất đẳng thức trở thành $$x^4+y^4+z^4+48(x^2+y^2+z^2)\ge 72xyz. \quad (2.9.2)$$

Giả sử $xy \ge 0$ rồi thay $z=-x-y$ vào (2.9.2), bất đẳng thức được viết lại dưới dạng

\[2(x^2+xy+y^2)^2+96(x^2+xy+y^2)+72xy(x+y) \ge 0.\]

Theo bất đẳng thức AM-GM, ta có

\[2(x^2+xy+y^2)^2+96(x^2+xy+y^2) \ge 18x^2y^2+72(x+y)^2,\]

lại có

\[x^2y^2+4(x+y)^2+4xy(x+y) = (2x+2y+xy)^2 \ge 0.\]
Đẳng thức xảy ra khi và chỉ khi $a=b=c$ hoặc $-3a=-3b=c.$ Chứng minh của chúng ta vì thế hoàn tất.


Nguyen Van Huyen
Ho Chi Minh City University Of Transport

#288
Nguyenhuyen_AG

Nguyenhuyen_AG

    Trung úy

  • Thành viên nổi bật 2016
  • 945 Bài viết
Bài 80 : Cho $x,y,z$ là các số thực thỏa mãn : $x+y+z=0$ và $x^{2}+y^{2}+z^{2}=6$. Tìm GTLN của : 

$P = \begin{vmatrix} (x-y)(y-z)(z-x) \end{vmatrix}$

 

Từ giả thiết ta suy ra $xy+yz+zx = -3.$ Đặt $p=x+y+z,\,q=xy+yz+zx,\,r=xyz,$ ta có

\[P^2 = (x-y)^2(y-z)^2(z-x)^2= p^2q^2-4q^3+2p(9q-2p^2)r-27r^2.\]

Do $p=0,$ và $q=-3,$ nên

\[P^2 = 54(2-r^2) \leqslant 54 \cdot 2 = 108.\]

Suy ra $P \leqslant 6\sqrt{3}.$ Đẳng thức xảy ra khi

\[ \left\{ \begin{aligned} & xyz = 0 \\& x+y+z=0 \\& {{x}^{2}}+{{y}^{2}}+{{z}^{2}}=6 \\
\end{aligned} \right.  \]

Giải hệ này ta được $(x,y,z) = \left ( 0,\sqrt{3},-\sqrt{3} \right ).$ Vậy $P_{\max} = 6\sqrt{3}.$

 

P/s. ducvipdh12 dạo này bận quá nên anh ít lên diễn đàn. GIờ đang được nghỉ gần 2 tuần nênh giành tí thời gian luyện lại bất đẳng thức, anh thây bên này có vẻ sôi nổi hơn bên Mathscope nhỉ, bên đấy vắng quá.


Bài viết đã được chỉnh sửa nội dung bởi Nguyenhuyen_AG: 16-06-2015 - 23:34

Nguyen Van Huyen
Ho Chi Minh City University Of Transport

#289
ducvipdh12

ducvipdh12

    Sĩ quan

  • Thành viên
  • 454 Bài viết

thời gian qua cảm ơn các bạn đã quan tâm và ủng hộ Topic ( đặc biệt cảm ơn sự trợ giúp gần đây của anh Huyện ) :D , mình hi vọng rằng là khi mà đạt được con số 200 bài viết thì mình sẽ nhờ BBT tổng hợp thành 1 ấn phẩm của VMF ( bên cạnh 2 ấn phẩm đã có của VMF ). Sắp tới mình sẽ mở thêm 2 hoặc 3 Topic nữa, nếu topic nào cũng hoạt động nhiệt thì VMF có thể trở lại thời kì huy hòang như xưa

Thân!


FAN THẦY THÔNG,ANH CẨN,THẦY VINH :icon6: :icon6:

#290
Nguyenhuyen_AG

Nguyenhuyen_AG

    Trung úy

  • Thành viên nổi bật 2016
  • 945 Bài viết

Bài 109 (IMO 2006). Tìm hằng số $M$ nhỏ nhất sao cho  bất đẳng thức

\[\left | ab(a^2-b^2)+bc(b^2-c^2)+ca(c^2-a^2) \right | \leqslant M(a^2+b^2+c^2)^2,\]

luôn đúng với mọi số thực $a,\,b,\,c$ thay đổi tùy ý.


Bài viết đã được chỉnh sửa nội dung bởi khanghaxuan: 18-06-2015 - 08:41

Nguyen Van Huyen
Ho Chi Minh City University Of Transport

#291
Hoang Nhat Tuan

Hoang Nhat Tuan

    Hỏa Long

  • Thành viên
  • 974 Bài viết

Bài xx (IMO 2006). Tìm hằng số $M$ nhỏ nhất sao cho  bất đẳng thức

\[\left | ab(a^2-b^2)+bc(b^2-c^2)+ca(c^2-a^2) \right | \leqslant M(a^2+b^2+c^2)^2,\]

luôn đúng với mọi số thực $a,\,b,\,c$ thay đổi tùy ý.

Chuẩn hóa $a^2+b^2+c^2=1$

Ta có: $\left | ab(a^2-b^2)+bc(b^2-c^2)+ca(c^2-a^2) \right |=\left |(a-b)(b-c)(c-a)(a+b+c) \right |\leq M$

Giả sử $c$ =min {a,b,c}

Áp dụng BĐT AM-GM: $\left [ 3(a^2+b^2+c^2) \right ]^2=\left [ 2(a-b)^2+(a+b+c)^2+2(a-c)(b-c) \right ]^2$

$\geq 4\left [ 2(a-b)^2+(a+b+c)^2 \right ].2(a-c)(b-c)\geq 16\sqrt{2}(a-b)(b-c)(a-c)(a+b+c)$

$=>(a-b)(b-c)(c-a)(a+b+c)\geq -\frac{9(a^2+b^2+c^2)^2}{16\sqrt{2}}=\frac{-9}{16\sqrt{2}}$

Do đó $M=\frac{9\sqrt{2}}{32}$

Spoiler


Ngài có thể trói cơ thể tôi, buộc tay tôi, điều khiển hành động của tôi: ngài mạnh nhất, và xã hội cho ngài thêm quyền lực; nhưng với ý chí của tôi, thưa ngài, ngài không thể làm gì được.

#292
dogsteven

dogsteven

    Đại úy

  • Thành viên
  • 1567 Bài viết

Bài xx (IMO 2006). Tìm hằng số $M$ nhỏ nhất sao cho  bất đẳng thức

\[\left | ab(a^2-b^2)+bc(b^2-c^2)+ca(c^2-a^2) \right | \leqslant M(a^2+b^2+c^2)^2,\]

luôn đúng với mọi số thực $a,\,b,\,c$ thay đổi tùy ý.

Đặt $p=a+b+c, q=ab+bc+ca, r=abc$. Không mất tính tổng quát, chuẩn hóa $p=1$. Khi đó bất đẳng thức trở thành:

$q^2-4q^3+2(9q-2)r-27r^2\leqslant M^2(1-2q)^4$

Cố định $q$, ta xét hàm số vế trái theo $r$, do đó hàm số này đạt cực đại tại $r=\dfrac{9q-2}{27}$, do đó ta cần phải có: $M^2\geqslant \dfrac{4(1-3q)^3}{27(1-2q)^4}=f(q)$

Để $M^2\geqslant f(q)$ với mọi $q$ khi và chỉ khi $M^2\geqslant \text{max}f(q)$

Ta có $f'(q)=\dfrac{4(1+6q)(1-3q)^2}{27(2q-1)^5}=0\Leftrightarrow q=\dfrac{1}{3}$ hoặc $q=\dfrac{-1}{6}$

Giờ thế giá trị ta được $M\geqslant \dfrac{9\sqrt{2}}{32}$


Quyết tâm off dài dài cày hình, số, tổ, rời rạc.


#293
Nguyenhuyen_AG

Nguyenhuyen_AG

    Trung úy

  • Thành viên nổi bật 2016
  • 945 Bài viết

Bài 55:(Ailen 2009)

Cho $a,b,c$ là các số thực thỏa $\left\{\begin{matrix} a+b+c=0\\a^2+b^2+c^2=1 \end{matrix}\right.$

Chứng minh rằng

$a^2b^2c^2\leq \frac{1}{54}$

 

 

Ngoài lời giải theo con đường khai triển $(a-b)^2(b-c)^2(c-a)^2 \geqslant 0,$ anh vừa tìm được một chứng minh tự nhiên hơn (lúc sáng đang xem Argentina đá chợt nghĩ ra :D)

 

Ta thấy bài toán là hệ quả của bất đẳng thức sau

\[(a^2+b^2+c^2)^3 \geqslant  54a^2b^2c^2, \quad (2)\]

Thật vậy theo bất đẳng thức Cauchy-Schwarz ta được

\[a^2+b^2+c^2 \geqslant \frac{(a+b)^2}{2}+c^2=\frac{3c^2}{2},\]

suy ra

\[(a^2+b^2+c^2)^3 \geqslant \frac{27c^6}{8}.\]

Như vậy ta chỉ cần chứng minh

\[\frac{27c^6}{8} \geqslant 54a^2b^2c^2,\]

hay

\[c^2 \geqslant 4\left | ab \right |.\]

Áp dụng bất đẳng thức AM-GM, ta có

\[4\left | ab \right | \leqslant \left |a+b  \right |^2 =(a+b)^2 = c^2.\]

Bài toán được chứng minh.

 

Nhận xét. Nếu thay $c=-a-b$ vào $(2)$ ta được

\[[a^2+b^2+(a+b)^2]^3 \geqslant  54a^2b^2(a+b)^2.\]

Ta có

\[[a^2+b^2+(a+b)^2]^3 - 54a^2b^2(a+b)^2=2(a-b)^2(2a+b)^2(2b+a)^2 \geqslant 0.\]

Đây cũng là một lời giải cho bài toán.

 


Bài viết đã được chỉnh sửa nội dung bởi Nguyenhuyen_AG: 17-06-2015 - 14:00

Nguyen Van Huyen
Ho Chi Minh City University Of Transport

#294
Belphegor Varia

Belphegor Varia

    Thượng sĩ

  • Thành viên
  • 227 Bài viết

Bài 96 (VMO 2001A ): Xét các số thực dương $x,y,z$ thoả mãn hệ điều kiện sau :

$1$ , $\frac{1}{\sqrt{2}}\leq z<\frac{\min (x\sqrt{2},y\sqrt{3})}{2}$
$2$ , $x+z\sqrt{3}\geq \sqrt{6}$
$3$ ,  $y\sqrt{3}+z\sqrt{10}\geq 2\sqrt{5}$

 

 

Hãy tìm giá trị lớn nhất của $P=\frac{1}{x^{2}}+\frac{2}{y^{2}}+\frac{3}{z^{2}}$

$\bigstar$ Từ điều kiện thứ nhất của đề bài suy ra : 

                 $\sqrt{2}\geq \frac{1}{z}> \frac{\sqrt{2}}{x}\Rightarrow \frac{1}{z^{2}}\leq 2$ và $\frac{z^{2}}{x^{2}}< \frac{1}{2}$
 

$\bigstar$ Từ điều kiện thứ hai của đề bài suy ra : 

                 $x^{2}+3z^{2}\geq 3\Rightarrow \frac{2}{3}+\frac{2z^{2}}{x^{2}}\geq \frac{2}{x^{2}}$

 

Do đó : $\frac{1}{x^{2}}+\frac{1}{z^{2}}=\frac{2}{x^{2}}+(\frac{1}{z^{2}}-\frac{1}{x^{2}})\leq \frac{2}{3}+\frac{2z^{2}}{x^{2}}+\frac{1}{z^{2}}.(1-\frac{z^{2}}{x^{2}})\leq \frac{8}{3}$

$\blacklozenge$ Tương tự , từ điều kiện thứ nhất và thứ ba ta chứng minh được 
                  $\frac{1}{y^{2}}+\frac{1}{z^{2}}\leq \frac{13}{5}$                   

Suy ra : $P=\frac{1}{x^{2}}+\frac{2}{y^{2}}+\frac{3}{z^{2}}=\frac{1}{x^{2}}+\frac{1}{z^{2}}+2(\frac{1}{y^{2}}+\frac{1}{z^{2}})\leq \frac{118}{15}$
 


Bài viết đã được chỉnh sửa nội dung bởi Belphegor Varia: 17-06-2015 - 14:51

$ \textbf{NMQ}$

Wait a minute, You have enough time. Also tomorrow will come 

Just take off her or give me a ride 

Give me one day or one hour or just one minute for a short word 

 


#295
Hoang Nhat Tuan

Hoang Nhat Tuan

    Hỏa Long

  • Thành viên
  • 974 Bài viết

Bài 76: ( Belarus 1998 ) Cho các số thực dương $a,b,c$. Chứng minh bất đẳng thức sau:

$\frac{a}{b}+\frac{b}{c}+\frac{c}{a}\geq \frac{a+b}{b+c}+\frac{b+c}{a+b}+1$

BĐT cần chứng minh tương đương:

$(\frac{a}{b}-\frac{a}{b+c})+(\frac{b}{c}-\frac{b}{b+c})+(\frac{c}{a}-\frac{c}{a+b})\geq \frac{b}{a+b}+1$

$<=>\frac{b^2}{c(b+c)}+\frac{ac}{b(b+c)}+\frac{bc}{a(a+b)}\geq \frac{a+2b}{a+b}$

Ta có:$\left [ \frac{b^2}{c(b+c){}}+\frac{ac}{b(b+c)}+\frac{bc}{a(a+b)} \right ]\left [ \frac{a(b+c)}{b}+(b+c)+(a+b) \right ]\geq (\sum \sqrt{\frac{ab}{c}})^2$

Mà: $\frac{a(b+c)}{b}+(b+c)+(a+b)=\frac{(a+b)(2b+c)}{b}$

Cần chứng minh:$(\sum \sqrt{\frac{ab}{c}})^2\geq \frac{(a+2b)}{a+b}.\frac{(a+b)(2b+c)}{b}=\frac{(a+2b)(2b+c)}{b}$

$<=>\frac{ab}{c}+\frac{bc}{a}\geq 2b$ (đúng theo BĐT AM-GM)

Vậy BĐT được chứng minh 


Ngài có thể trói cơ thể tôi, buộc tay tôi, điều khiển hành động của tôi: ngài mạnh nhất, và xã hội cho ngài thêm quyền lực; nhưng với ý chí của tôi, thưa ngài, ngài không thể làm gì được.

#296
dangkhuong

dangkhuong

    Sĩ quan

  • Thành viên
  • 312 Bài viết

Bài 110: Bài toán sau khá hay mong các bạn thử sức :ukliam2:  :ukliam2:  :ukliam2:

 

Cho a,b,c>0. Cmr:

   

           $\frac{a^3+2}{(b+c)^2}+\frac{b^3+2}{(c+a)^2}+\frac{c^3+2}{(a+b)^2}\geq \frac{3(a+b+c)}{4}$             

 

 với $a^2+b^2+c^2=3$


Bài viết đã được chỉnh sửa nội dung bởi khanghaxuan: 18-06-2015 - 08:44

:ukliam2:  :ukliam2:  :ukliam2:


#297
ducvipdh12

ducvipdh12

    Sĩ quan

  • Thành viên
  • 454 Bài viết

Bài ???: Bài toán sau khá hay mong các bạn thử sức :ukliam2:  :ukliam2:  :ukliam2:

 

Cho a,b,c>0. Cmr:

   

           $\frac{a^3+2}{(b+c)^2}+\frac{b^3+2}{(c+a)^2}+\frac{c^3+2}{(a+b)^2}\geq \frac{3(a+b+c)}{4}$             

 

 với $a^2+b^2+c^2=3$

Nhắc nhở đầu tiên là bạn phải ghi rõ bài số mấy,còn nhắc nhở thứ 2 thì ghi rõ nguồn,nếu không phải xuất phải từ 1 kì thi MO các nước thì mình sẽ ẩn bài của bạn khỏi topic


FAN THẦY THÔNG,ANH CẨN,THẦY VINH :icon6: :icon6:

#298
an1712

an1712

    Trung sĩ

  • Thành viên
  • 149 Bài viết

Bài ???: Bài toán sau khá hay mong các bạn thử sức :ukliam2:  :ukliam2:  :ukliam2:

 

Cho a,b,c>0. Cmr:

   

           $\frac{a^3+2}{(b+c)^2}+\frac{b^3+2}{(c+a)^2}+\frac{c^3+2}{(a+b)^2}\geq \frac{3(a+b+c)}{4}$             

 

 với $a^2+b^2+c^2=3$

ta có: $a^3+2\geq \frac{3}{2}a^2+\frac{3}{2}\geq \frac{3}{4}(a+1)^2$

bdt$\Leftrightarrow \sum (\frac{a+1}{b+c})^2\geq a+b+c$

áp dụng cô si:$(\frac{a+1}{b+c})^2+\frac{b+c}{2}+\frac{(a+1)(b+c)}{4}\geq \frac{3}{2}(a+1)$

tương tự vs 2 biểu thức còn lại ta đc:

$\sum (\frac{a+1}{b+c})^2\geq \frac{9-(ab+bc+ac)}{2}\geq \frac{9-\frac{(a+b+c)^2}{3}}{2}$

đặt a+b+c=t >0 

bdt$\Leftrightarrow (t+9)(3-t)\geq 0$ (luôn đúng)


Bài viết đã được chỉnh sửa nội dung bởi an1712: 17-06-2015 - 21:33

tiến tới thành công  :D


#299
dangkhuong

dangkhuong

    Sĩ quan

  • Thành viên
  • 312 Bài viết

Bài 110: Bài toán sau khá hay mong các bạn thử sức :ukliam2:  :ukliam2:  :ukliam2:

 

Cho a,b,c>0. Cmr:

   

           $\frac{a^3+2}{(b+c)^2}+\frac{b^3+2}{(c+a)^2}+\frac{c^3+2}{(a+b)^2}\geq \frac{3(a+b+c)}{4}$             

 

 với $a^2+b^2+c^2=3$

bài trên có lời giải như sau:(cách khác bạn an1712)

 

Áp dụng trực tiếp bđt AM-GM cho 3 số thì việc cm bđt trên quy về 2 bđt sau:

 1)$(a^3+2)(b^3+2)(c^3+2)\geq (a+b+c)^3$

 2)$\sqrt[3]{(a+b)^2(b+c)^2(c+a)^2}\leq 4$

Bđt đầu thì dễ thấy rằng là đúng theo bđt Holder. 

Bđt sau thì cũng đúng theo AM-GM và giả thuyết từ đó ta có đpcm. :ukliam2:  :ukliam2:  :ukliam2: 


Bài viết đã được chỉnh sửa nội dung bởi dangkhuong: 19-06-2015 - 15:38

:ukliam2:  :ukliam2:  :ukliam2:


#300
dangkhuong

dangkhuong

    Sĩ quan

  • Thành viên
  • 312 Bài viết

Bài 111(Greece MO): Cho a,b,c là các số thực dương sao cho ab+bc+ca=1 . Cmr:

$\sqrt{\frac{a^2+1}{b^2+6bc+c^2}}+\sqrt{\frac{b^2+1}{c^2+6ca+a^2}}+\sqrt{\frac{c^2+1}{a^2+6ab+b^2}}\geq \frac{3\sqrt{2}}{2}$

                           


Bài viết đã được chỉnh sửa nội dung bởi khanghaxuan: 18-06-2015 - 18:33

:ukliam2:  :ukliam2:  :ukliam2:





1 người đang xem chủ đề

0 thành viên, 1 khách, 0 thành viên ẩn danh